Preptest12 secion 4(LR) #24 help pls!!!!!!!!!!!!! Forum

Prepare for the LSAT or discuss it with others in this forum.
Post Reply
wejuhn

New
Posts: 11
Joined: Thu Jan 14, 2010 3:30 am

Preptest12 secion 4(LR) #24 help pls!!!!!!!!!!!!!

Post by wejuhn » Sat Jul 17, 2010 6:25 pm

Although I answered this question correctly, by guessing, I don't know how to solve it or explain to myself why it's correct.
Can anybody help pls?????
I would truly appreciate it!!!!

User avatar
DrackedaryMaster

Bronze
Posts: 180
Joined: Sun Jul 04, 2010 5:11 pm

Re: Preptest12 secion 4(LR) #24 help pls!!!!!!!!!!!!!

Post by DrackedaryMaster » Sat Jul 17, 2010 9:17 pm

The main rule to this one is that once flasks are mixed together, the product can't be used for any additional experiment.

If one of the flasks after the two experiments were completed is Orange. This means, based on rules, that Flasks 2 & 3 were apart of either the first or second experiment, because mixing 2&3 = Orange. Since the product of an experiment cannot be used for another experiment, it follows that the other experiment had to be flaks 1&4 which = Green.

Hope this helps.

wejuhn

New
Posts: 11
Joined: Thu Jan 14, 2010 3:30 am

Re: Preptest12 secion 4(LR) #24 help pls!!!!!!!!!!!!!

Post by wejuhn » Sun Jul 18, 2010 4:15 pm

Thanx,but my question was on lr #24.....
I appreiciat ur effort though.Thank you

reallyemo

New
Posts: 5
Joined: Sun Jul 11, 2010 7:59 pm

Re: Preptest12 secion 4(LR) #24 help pls!!!!!!!!!!!!!

Post by reallyemo » Mon Jul 19, 2010 8:17 pm

What the stimulus says is essentially that people making economic decisions can't (readily) compare economic factors with other costs and benefits. It then says that, to be able to compare them, you need to assign monetary values to the environmental factors. However, to find the monetary value of something, you have to compare it's costs and benefits (which is what people can't do in the first place). So the argument concludes that what these people are trying to do is hindered by what it's trying to do (that's a bit confusing, but it essentially just concludes that their aim is cyclical in that you need one thing to get another but to get the first thing you need the second).

The question stem asks about the passage's conclusion. The answer is (a) because, assuming these considerations are true, then the process does seem to be very cyclical, and thus the conclusion is strong. However, the argument does assume that the only way for monetary values for environmental factors to be assigned is by people (because if they could be determined by a computer program then the program could decide the monetary values and then people would be able to compare the costs and benefits without it being very cyclical) For the argument to be cyclical as the conclusion claims, people making economic decisions about these factors can be the only way that we can get monetary values.

Does that make sense?

wejuhn

New
Posts: 11
Joined: Thu Jan 14, 2010 3:30 am

Re: Preptest12 secion 4(LR) #24 help pls!!!!!!!!!!!!!

Post by wejuhn » Mon Jul 19, 2010 11:53 pm

Wow!!!! Thank u very very much for the kind and clear explanation!! I see the point of the question now. I truly appreciate it, it helped me a lot!!!!!!!!!! Thank you

Want to continue reading?

Register now to search topics and post comments!

Absolutely FREE!


Post Reply

Return to “LSAT Prep and Discussion Forum”